30 bài tập về Giá trị lượng giác của một góc bất kỳ từ 0 độ đến 180 độ

Làm đề thi

Câu hỏi 1 :

Trong các đẳng thức sau đây, đẳng thức nào đúng?

  • A \(\sin {150^0} =  - {{\sqrt 3 } \over 2}\)
  • B \(\cos {150^0} = {{\sqrt 3 } \over 2}\)
  • C \(\tan {150^0} =  - {1 \over {\sqrt 3 }}\)
  • D \(\cot {150^0} = \sqrt 3 \)

Đáp án: C

Phương pháp giải:

Sử dụng bảng giá trị lượng giác các góc đặc biệt.

Lời giải chi tiết:

Vì \(\alpha  = {150^0}\)  là góc tù nên \(\sin \alpha  > 0,\cos \alpha  < 0,\tan \alpha  = {{\sin \alpha } \over {c{\rm{os}}\alpha }} < 0,\cot \alpha  < 0.\)

Do đó các đáp án A, B, D đều sai. Ta chỉ xét đáp án C. Ta có \(\tan {150^ \circ } =  - \tan {30^ \circ } =  - {1 \over {\sqrt 3 }}.\)

Chọn C.

Đáp án - Lời giải

Câu hỏi 2 :

Tính giá trị biểu thức \(P = \sin {30^0}\cos {60^0} + \sin {60^0}\cos {30^0}.\)

  • A  \(P = 1.\)                                  
  • B  \(P = 0.\)                                  
  • C  \(P = \sqrt 3 .\)                      
  • D  \(P =  - \sqrt 3 .\)

Đáp án: A

Phương pháp giải:

Tính giá trị của biểu thức.

Lời giải chi tiết:

\(P = \sin {30^0}\cos {60^0} + \sin {60^0}\cos {30^0}. = \dfrac{1}{2}.\dfrac{1}{2} + \dfrac{{\sqrt 3 }}{2}.\dfrac{{\sqrt 3 }}{2} = \dfrac{1}{4} + \dfrac{3}{4} = 1\)

Chọn A.

Đáp án - Lời giải

Câu hỏi 3 :

Góc \(\frac{{7\pi }}{6}\) có số đo bằng độ là:

  • A \({30^o}\)                              
  • B \({105^o}\)                          
  • C \({150^o}\)
  • D \({210^o}\)

Đáp án: D

Phương pháp giải:

\(\pi  = {180^o}.\) 

Lời giải chi tiết:

\(\frac{{7\pi }}{6} = \frac{{{{7.180}^o}}}{6} = {210^o}\)

Chọn D.

Đáp án - Lời giải

Câu hỏi 4 :

Mệnh đề nào sau đây là đúng?

  • A \(1\,rad = {1^0}\)              
  • B \({1^0} = \frac{1}{\pi }\)
  • C \(\pi \,rad = {180^0}\)    
  • D \(\pi \,rad = {\left( {\frac{1}{{180}}} \right)^0}\).

Đáp án: C

Phương pháp giải:

\(\pi \) rad \( = {180^o}\)

Lời giải chi tiết:

Ta có: \(\pi \) rad \( = {180^o}\)

Vậy C đúng

Chọn C.

Đáp án - Lời giải

Câu hỏi 5 :

Số đo theo đợn vị radian của góc \({315^o}\) là :

  • A \(\frac{{7\pi }}{2}\)                  
  • B \(\frac{{7\pi }}{4}\)
  • C \(\frac{{2\pi }}{7}\)
  • D \(\frac{{4\pi }}{7}\)

Đáp án: B

Phương pháp giải:

\(\pi  = {180^o}\)

Lời giải chi tiết:

\({315^o} = \frac{{315\pi }}{{180}} = \frac{{7\pi }}{4}\)

Chọn B.

Đáp án - Lời giải

Câu hỏi 6 :

Giá trị của biểu thức \(P = c{\rm{os3}}{{\rm{0}}^o}{\rm{cos6}}{{\rm{0}}^o} - \sin {30^o}\sin {60^o}?\)

  • A \(P = \sqrt 3 \)                          
  • B \(P = \frac{{\sqrt 3 }}{2}\)                               
  • C \(P = 1\)                                  
  • D \(P = 0\)

Đáp án: D

Phương pháp giải:

Nắm được bảng giá trị lượng giác của một số góc đặc biệt như \({30^o};{45^o};{60^o};{90^o};...\)

Cách 2: Với  \(\angle B + \angle C = {90^o} \Rightarrow \left\{ \begin{array}{l}\sin \angle B = \cos \angle C\\\cos \angle B = \sin \angle C\end{array} \right..\)

Lời giải chi tiết:

Ta có: \(P = \cos {30^o}.\cos {60^o} - \sin {30^o}.\sin {60^o} = \frac{{\sqrt 3 }}{2}.\frac{1}{2} - \frac{1}{2}.\frac{{\sqrt 3 }}{2} = 0\)

Cách 2: \(P = \cos {30^o}.\cos {60^o} - \sin {30^o}.\sin {60^o} = \cos {30^o}.\cos {60^o} - \cos {60^o}.\cos {30^0} = 0.\)

Chọn  D.

Đáp án - Lời giải

Câu hỏi 7 :

Trong tam giác \(ABC\) vuông ở \(A\) có góc \(\angle B = {30^o}.\) Khẳng định nào sau đây là sai?

  • A \(\tan C = \frac{1}{{\sqrt 3 }}\)                
  • B \(\sin C = \frac{{\sqrt 3 }}{2}\)            
  • C \(\cos C = \frac{1}{2}\)
  • D \(\cot C = \frac{1}{{\sqrt 3 }}\)

Đáp án: A

Phương pháp giải:

Cho tam giác \(ABC\) vuông ở \(A.\)  Khi đó, \(\angle B + \angle C = {90^o} \Rightarrow \left\{ \begin{array}{l}\sin \angle B = \cos \angle C\\\cos \angle B = \sin \angle C\end{array} \right.\)

Nắm được giá trị lượng giác của một số góc đặc biệt như \({30^o};\,\,{45^o};\,\,\,{60^o};\,\,{90^o};.....\)

Lời giải chi tiết:

\(\Delta ABC\) vuông tại \(A \Rightarrow \angle B + \angle C = {90^0} \Rightarrow \angle C = {90^0} - \angle B = {90^0} - {30^0} = {60^0}.\)

\( \Rightarrow \left\{ \begin{array}{l}\sin \angle C = \sin {60^0} = \frac{{\sqrt 3 }}{2};\,\,\,\cos \angle C = \cos {60^0} = \frac{1}{2}\\\tan \angle C = \tan {60^0} = \sqrt 3 ;\,\,\,\cot \angle C = \cot {60^0} = \frac{1}{{\sqrt 3 }}\end{array} \right..\)

Chọn A.

Đáp án - Lời giải

Câu hỏi 8 :

Cho \(\alpha \) là góc tù. Khẳng định nào sau đây là sai?

  • A \(\cos \alpha  < 0\)    
  • B    \(\sin \alpha  < 0\)
  • C \(\tan \alpha  < 0\)  
  • D \(\cot \alpha  < 0\)

Đáp án: B

Phương pháp giải:

Khi góc \(\alpha \) tù thì  \(\sin \alpha  > 0,\cos \alpha  < 0,\tan \alpha  < 0,\cot \alpha  < 0.\) 

Lời giải chi tiết:

Khi góc \(\alpha \) tù thì \(\sin \alpha  > 0,\cos \alpha  < 0,\tan \alpha  < 0,\cot \alpha  < 0.\) 

Chọn B.

Đáp án - Lời giải

Câu hỏi 9 :

Cho hệ thức đúng được suy ra từ hệ thức \({\rm{co}}{{\rm{s}}^2}\alpha  + {\sin ^2}\alpha  = 1?\)

  • A \({\rm{co}}{{\rm{s}}^2}\frac{\alpha }{2} + {\sin ^2}\frac{\alpha }{2} = \frac{1}{2}\)
  • B \({\rm{co}}{{\rm{s}}^2}\frac{\alpha }{3} + {\sin ^2}\frac{\alpha }{3} = \frac{1}{3}\)
  • C \({\rm{co}}{{\rm{s}}^2}\frac{\alpha }{4} + {\sin ^2}\frac{\alpha }{4} = \frac{1}{4}\)
  • D \(5\left( {{\rm{co}}{{\rm{s}}^2}\frac{\alpha }{5} + {{\sin }^2}\frac{\alpha }{5}} \right) = 5\)\(\)

Đáp án: D

Phương pháp giải:

Với một góc \(\beta \) bất kì, ta luôn có: \({\rm{co}}{{\rm{s}}^2}\beta  + {\sin ^2}\beta  = 1.\)

Lời giải chi tiết:

Từ biểu thức: \({\rm{co}}{{\rm{s}}^2}\alpha  + {\sin ^2}\alpha  = 1,\) ta suy ra \({\rm{co}}{{\rm{s}}^2}\frac{\alpha }{5} + {\sin ^2}\frac{\alpha }{5} = 1 \Rightarrow 5\left( {{\rm{co}}{{\rm{s}}^2}\frac{\alpha }{5} + {{\sin }^2}\frac{\alpha }{5}} \right) = 5\)

Chọn  D.

Đáp án - Lời giải

Câu hỏi 10 :

Cho hai góc \(\alpha ,\beta \) thỏa mãn \(\alpha  < \beta \) và \(90^\circ  < \alpha ,\beta  < 180^\circ .\) Tìm khẳng định đúng?

  • A \(\cos \alpha  > \cos \beta .\)
  • B \(\tan \alpha  + \cot \beta  > 0.\)         
  • C \(\cot \alpha .\tan \beta  < 0.\)
  • D \(\sin \alpha  < \sin \beta .\)

Đáp án: A

Phương pháp giải:

Sử dụng giá trị lượng giác của một cung.

Lời giải chi tiết:

Với \(\alpha  < \beta \) và \(90^\circ  < \alpha ,\beta  < 180^\circ \) thì \(\cos \alpha  > \cos \beta .\)

Chọn A.

Đáp án - Lời giải

Câu hỏi 11 :

Cho góc \(0^\circ  < \alpha  < 90^\circ .\) Khẳng định nào sau đây sai?

  • A \(\tan \alpha  > 0.\)
  • B \(\cos \alpha  < 0.\)
  • C \(\sin \alpha  > 0.\)          
  • D \(\cot \alpha  > 0.\)

Đáp án: B

Phương pháp giải:

Sử dụng đường tròn lượng giác để đánh giá các giá trị lượng giác của một góc.

Lời giải chi tiết:

Với \(0^\circ  < \alpha  < 90^\circ \) thì \(\cos \alpha  > 0.\)

Chọn B.

Đáp án - Lời giải

Câu hỏi 12 :

Giá trị của\(\tan \frac{\pi }{6}\) là

  • A \(\frac{{\sqrt 3 }}{3}\).
  • B \(-\frac{{\sqrt 3 }}{3}\).  
  • C \(\sqrt 3 \).  
  • D \( - \sqrt 3 \).

Đáp án: A

Phương pháp giải:

Bấm máy tính hoặc dùng bảng lượng giác.

Lời giải chi tiết:

Ta có: \(\tan \frac{\pi }{6} = \frac{{\sqrt 3 }}{3}\)

Chọn A.

Đáp án - Lời giải

Câu hỏi 13 :

Biết \(\cot \alpha  = 3\), khi đó giá trị của \(\sin \left( {2\alpha  - \frac{\pi }{4}} \right)\) là:

  • A

     \( - \frac{{\sqrt 2 }}{2}\)                         

  • B

     \(\frac{{\sqrt 2 }}{2}\)                             

  • C

    \(\frac{{ - \sqrt 2 }}{{10}}\)                                 

  • D  \(\frac{{\sqrt 2 }}{{10}}\)

Đáp án: C

Phương pháp giải:

 

Sử dụng các công thức sau đây:

\(\sin \left( {a - b} \right) = \sin a\cos b - \sin b\cos a;\,\,\frac{1}{{{{\sin }^2}a}} = 1 + {\cot ^2}a;\,\,\cos 2a = 1 - 2{\sin ^2}a;\,\,\cot 2a = \frac{{{{\cot }^2}a - 1}}{{2\cot a}}\)

Lời giải chi tiết:

\(\begin{array}{l}\sin \left( {2\alpha  - \frac{\pi }{4}} \right) = \sin 2\alpha \cos \frac{\pi }{4} - \cos 2\alpha \sin \frac{\pi }{4} = \frac{{\sqrt 2 }}{2}\left( {\sin 2\alpha  - \cos 2\alpha } \right)\\\cot \alpha  = 3 \Rightarrow \frac{1}{{{{\sin }^2}\alpha }} = 1 + {\cot ^2}\alpha  = 1 + {3^2} = 10\\ \Rightarrow \cos 2\alpha  = 1 - 2{\sin ^2}\alpha  = 1 - 2.\frac{1}{{10}} = \frac{4}{5}\\\cot 2\alpha  = \frac{{{{\cot }^2}\alpha  - 1}}{{2\cot \alpha }} = \frac{4}{3} \Rightarrow \sin 2\alpha  = \frac{{\cos 2\alpha }}{{\cot 2\alpha }} = \frac{3}{5}\\ \Rightarrow \sin \left( {2\alpha  - \frac{\pi }{4}} \right) = \frac{{\sqrt 2 }}{2}\left( {\frac{3}{5} - \frac{4}{5}} \right) = \frac{{ - \sqrt 2 }}{{10}}\end{array}\)

Chọn C.

Đáp án - Lời giải

Câu hỏi 14 :

Cho tam giác ABC vuông tại A, góc B bằng 300. Khẳng định nào sau đây là sai?

  • A \({\rm{cos B  =  }}{1 \over {\sqrt 3 }}\)
  • B \({\rm{sin C  =  }}{{\sqrt 3 } \over 2}\)
  • C \({\rm{cos C  =  }}{1 \over 2}\)
  • D \({\rm{sin B  =  }}{1 \over 2}\)

Đáp án: A

Phương pháp giải:

Sử dụng bảng giá trị lượng giác các góc đặc biệt.

Lời giải chi tiết:

Tam giác ABC vuông tại A, \(\widehat B = {30^0} \Rightarrow \widehat C = {90^0} - {30^0} = {60^0}.\)

Theo bảng giá trị lượng giác các góc đặc biệt

\(\eqalign{  & \sin B = \sin {30^0} = \cos C = \cos {60^0} = {1 \over 2}  \cr   & \sin C = \sin 60 = \cos B = \cos 30 = {{\sqrt 3 } \over 2}. \cr} \)

Vậy đáp án A sai.

Chọn A.

Đáp án - Lời giải

Câu hỏi 15 :

Giá trị  \(\sin \frac{{47\pi }}{6}\) bằng:

  • A \( - \frac{{\sqrt 3 }}{2}\). 
  • B \(\frac{1}{2}\).                
  • C \(\frac{{\sqrt 2 }}{2}\). 
  • D \( - \frac{1}{2}\).

Đáp án: D

Phương pháp giải:

\(\sin \left( {x + k2\pi } \right) = \sin x\)

Lời giải chi tiết:

\(\sin \frac{{47\pi }}{6} = \sin \left( {8\pi  - \frac{\pi }{6}} \right) = \sin \left( { - \frac{\pi }{6}} \right) =  - \sin \frac{\pi }{6} =  - \frac{1}{2}\)

Chọn D.

Đáp án - Lời giải

Câu hỏi 16 :

Cung có số đo 2250  được đổi sang số đo rad là :

  • A \(225\pi \).       
  • B \(\frac{{3\pi }}{4}\).                    
  • C \(\frac{{5\pi }}{4}\).          
  • D \(\frac{{4\pi }}{3}\).                                  

Đáp án: C

Phương pháp giải:

\(\pi \) rad \( = {180^o}\)

Lời giải chi tiết:

\({225^o} = \frac{{225\pi }}{{180}} = \frac{{5\pi }}{4}\)

Chọn C.

Đáp án - Lời giải

Câu hỏi 17 :

Một đường tròn có bán kính \(R = 75cm\). Độ dài của cung trên đường tròn đó có số đo \(\alpha  = \frac{\pi }{{25}}\) là:

  • A \(3\pi \,\,cm\)                        
  • B \(4\pi \,\,cm\)                        
  • C \(5\pi \,\,cm\)
  • D \(6\pi \,\,cm\)

Đáp án: A

Phương pháp giải:

Độ dài cung = bán kính \( \times n.\)  Trong đó n là số đo góc chứa cung tính theo radian.

Lời giải chi tiết:

Độ dài cung có số đo \(\alpha  = \frac{\pi }{{25}}\)  là: \(R.\alpha  = 75.\frac{\pi }{{25}} = 3\pi \;\;\left( {cm} \right).\)

Chọn A.

Đáp án - Lời giải

Câu hỏi 18 :

Cho \(\sin \alpha  = \frac{1}{3}.\)Tính giá trị biểu thức \(P = 3{\sin ^2}\alpha  + {\cos ^2}\alpha .\)

  • A \(P = \frac{{25}}{9}.\)                         
  • B \(P = \frac{9}{{25}}.\)
  • C \(P = \frac{{11}}{9}.\)
  • D \(P = \frac{9}{{11}}.\)             

Đáp án: C

Phương pháp giải:

Ta sử dụng công thức: \({\sin ^2}\alpha  + {\cos ^2}\alpha  = 1.\) 

Lời giải chi tiết:

Ta có: \(P = 3{\sin ^2}\alpha  + {\cos ^2}\alpha  = 3{\sin ^2}\alpha  + \left( {1 - {{\sin }^2}\alpha } \right) = 2{\sin ^2}\alpha  + 1 = 2{\left( {\frac{1}{3}} \right)^2} + 1 = \frac{{11}}{9}.\)

Chọn  C.

Đáp án - Lời giải

Câu hỏi 19 :

Tam giác đều \(ABC\) có đường cao \(AH.\) Khẳng định nào sau đây là đúng?

  • A \(\sin \angle BAH = \frac{{\sqrt 3 }}{2}\)                  
  • B \(\sin \angle ABC = \frac{{\sqrt 3 }}{2}\)                 
  • C \(\cos \angle BAH = \frac{1}{{\sqrt 3 }}\)       
  • D \(\sin \angle AHC = \frac{1}{2}\)

Đáp án: B

Phương pháp giải:

Xác định số đo các góc, từ đó tính giá trị lượng giác của góc đã cho.

Lời giải chi tiết:

\(\Delta ABC\) là tam giác đều có đường cao \(AH \Rightarrow AH\) cùng là đường phân giác của \(\Delta ABC\)

\( \Rightarrow \angle BAH = \angle HAC = {30^o} \Rightarrow \left\{ {\begin{array}{*{20}{c}}{\sin \angle BAH = \sin \angle HAC = \frac{1}{2}}\\{{\rm{cos}}\angle BAH = \frac{{\sqrt 3 }}{2}}\end{array} \Rightarrow } \right.\) loại A, C.

Vì \(\angle ABC = {60^o} \Rightarrow \sin \angle ABC = \frac{{\sqrt 3 }}{2}.\)

Chọn  B

Đáp án - Lời giải

Câu hỏi 20 :

Cho \(\cos 15^\circ  = \frac{{\sqrt 2  + \sqrt 6 }}{4}.\) Khẳng định nào sau đây đúng?

  • A \(\cos 75^\circ  = \frac{{\sqrt 2  + \sqrt 6 }}{4}.\)
  • B \(\cos 165^\circ  =  - \frac{{\sqrt 2  + \sqrt 6 }}{4}.\)                      
  • C \(\cos 165^\circ  = \frac{{\sqrt 2  - \sqrt 6 }}{4}.\)
  • D \(\sin 75^\circ  =  - \frac{{\sqrt 2  + \sqrt 6 }}{4}.\)

Đáp án: B

Phương pháp giải:

Áp dụng công thức: \(\cos \left( {\pi  - x} \right) =  - \cos x.\)

Lời giải chi tiết:

Ta có: \(\cos 165^\circ  =  - \cos \left( {180^\circ  - 165^\circ } \right) =  - \cos 15^\circ  =  - \frac{{\sqrt 2  + \sqrt 6 }}{4}.\)

Chọn B.

Đáp án - Lời giải

Câu hỏi 21 :

Cho \(O\) là tâm đường tròn ngoại tiếp tam giác đều \(MNP.\) Góc nào sau đây bằng \({120^o}?\)

  • A \(\left( {\overrightarrow {MN} ,\overrightarrow {MP} } \right)\)
  • B \(\left( {\overrightarrow {MN} ,\overrightarrow {NP} } \right)\)
  • C \(\left( {\overrightarrow {MO} ,\overrightarrow {ON} } \right)\)
  • D \(\left( {\overrightarrow {MN} ,\overrightarrow {OP} } \right)\) 

Đáp án: B

Phương pháp giải:

Cho hai vecto \(\overrightarrow a ,\,\,\overrightarrow b  \ne \overrightarrow 0 .\)  Từ \(O,\) vẽ \(\overrightarrow {OA}  = \overrightarrow a ,\,\,\overrightarrow {OB}  = \overrightarrow b  \Rightarrow \angle AOB = \angle \left( {\overrightarrow a ,\,\,\overrightarrow b } \right).\)

Lời giải chi tiết:

Vẽ \(\overrightarrow {NE}  = \overrightarrow {MN} .\) Khi đó \(\left( {\overrightarrow {MN} ,\overrightarrow {NP} } \right) = \left( {\overrightarrow {NE} ,\overrightarrow {NP} } \right) = \angle PNE = {180^o} - \angle MNP = {180^o} - {60^o} = {120^o}.\)                            

Vẽ \(\overrightarrow {OF}  = \overrightarrow {MO}  \Rightarrow \left( {\overrightarrow {MO} ,\overrightarrow {ON} } \right) = \left( {\overrightarrow {OF} ,\overrightarrow {ON} } \right) = \angle NOF = {60^o}\)

Vì  \(MN \bot OP\,\, \Rightarrow \left( {\overrightarrow {MN} ,\overrightarrow {OP} } \right) = {90^o}\)

Ta có \(\left( {\overrightarrow {MN} ,\overrightarrow {MP} } \right) = \angle NMP = {60^o}.\)

Chọn  B.

Đáp án - Lời giải

Câu hỏi 22 :

Cho biết \(\cot \alpha  = 5.\) Giá trị của \(A = 2{\cos ^2}\alpha  + 5\sin \alpha \cos \alpha  + 1\)bằng bao nhiêu?

  • A \(A = \frac{{10}}{{26}}.\)
  • B \(A = \frac{{100}}{{26}}.\)      
  • C \(A = \frac{{101}}{{26}}.\)      
  • D \(A = \frac{{50}}{{26}}.\)

Đáp án: C

Phương pháp giải:

Ta sử dụng công thức sau: \(\tan \alpha  = \frac{{\sin \alpha }}{{\cos \alpha }};\,\,\,\,1 + {\cot ^2}\alpha  = \frac{1}{{{{\sin }^2}\alpha }}.\)

Lời giải chi tiết:

Ta có:

\(\begin{array}{l}A = 2{\cos ^2}\alpha  + 5\sin \alpha \cos \alpha  + 1 = {\sin ^2}\alpha \left( {2\frac{{{{\cos }^2}\alpha }}{{{{\sin }^2}\alpha }} + 5\frac{{\cos \alpha }}{{\sin \alpha }} + \frac{1}{{{{\sin }^2}\alpha }}} \right)\\\,\,\,\,\, = \frac{1}{{1 + {{\cot }^2}\alpha }}\left( {2{{\cot }^2}\alpha  + 5\cot \alpha  + 1 + {{\cot }^2}\alpha } \right)\\\,\,\,\,\, = \frac{{3{{\cot }^2}\alpha  + 5\cot \alpha  + 1}}{{{{\cot }^2}\alpha  + 1}} = \frac{{{{3.5}^2} + 5.5 + 1}}{{{5^2} + 1}} = \frac{{101}}{{26}}.\end{array}\)

Chọn  C

Đáp án - Lời giải

Câu hỏi 23 :

Cho biết \(2\cos \alpha  + \sqrt 2 \sin \alpha  = 2,{0^o} < \alpha  < {90^o}.\)  Tính giá trị của \(\cot \alpha .\)

  • A \(\cot \alpha  = \frac{{\sqrt 5 }}{4}.\)   
  • B \(\cot \alpha  = \frac{{\sqrt 2 }}{4}.\)               
  • C \(\cot \alpha  = \frac{{\sqrt 3 }}{4}.\)
  • D \(\cot \alpha  = \frac{{\sqrt 2 }}{2}.\)

Đáp án: B

Phương pháp giải:

Sử dụng công thức: \(\cot \alpha  = \frac{{\cos \alpha }}{{\sin \alpha }};\,\,\,{\cos ^2}\alpha  + {\sin ^2}\alpha  = 1.\)

Lời giải chi tiết:

Ta có: \(2\cos \alpha  + \sqrt 2 \sin \alpha  = 2 \Leftrightarrow \sqrt 2 \sin \alpha  = 2 - 2\cos \alpha \)

       \(\begin{array}{l} \Leftrightarrow 2{\sin ^2}\alpha  = {\left( {2 - 2\cos \alpha } \right)^2}\\ \Leftrightarrow 2 - 2{\cos ^2}\alpha  = 4 - 8\cos \alpha  + 4{\cos ^2}\alpha \\ \Leftrightarrow 6{\cos ^2}\alpha  - 8\cos \alpha  + 2 = 0 \Leftrightarrow \left[ {\begin{array}{*{20}{c}}{\cos \alpha  = 1}\\{\cos \alpha  = \frac{1}{3}}\end{array}} \right.\end{array}\)

+ \(\cos \alpha  = 1:\) không thoả mãn vì \({0^o} < \alpha  < {90^o}.\)

+ \(\cos \alpha  = \frac{1}{3} \Rightarrow \sin \alpha  = \frac{{2\sqrt 2 }}{3} \Rightarrow \cot \alpha  = \frac{{\cos \alpha }}{{\sin \alpha }} = \frac{{\sqrt 2 }}{4}.\)

Chọn  B.

Đáp án - Lời giải

Câu hỏi 24 :

Tam giác \(ABC\) có góc \(\angle A = {100^o}\) và  có trực tâm \(H.\)

Tính tổng \(\left( {\overrightarrow {HA} ,\overrightarrow {HB} } \right) + \left( {\overrightarrow {HB} ,\overrightarrow {HC} } \right) + \left( {\overrightarrow {HC} ,\overrightarrow {HA} } \right).\)

  • A \({360^o}\)  
  • B \({160^o}\)      
  • C \({100^o}\)                  
  • D \({80^o}\)

Đáp án: B

Phương pháp giải:

Cho hai vecto \(\overrightarrow a ,\,\,\overrightarrow b  \ne \overrightarrow 0 .\)  Từ \(O,\) vẽ \(\overrightarrow {OA}  = \overrightarrow a ,\,\,\overrightarrow {OB}  = \overrightarrow b  \Rightarrow \angle AOB = \angle \left( {\overrightarrow a ,\,\,\overrightarrow b } \right).\)

Lời giải chi tiết:

Ta có: \(\left\{ {\begin{array}{*{20}{c}}{\left( {\overrightarrow {HA} ,\overrightarrow {HB} } \right) = \angle BHA}\\{\left( {\overrightarrow {HB} ,\overrightarrow {HC} } \right) = \angle BHC}\\{\left( {\overrightarrow {HC} ,\overrightarrow {HA} } \right) = \angle CHA}\end{array}} \right.\)

\( \Rightarrow \left( {\overrightarrow {HA} ,\overrightarrow {HB} } \right) + \left( {\overrightarrow {HB} ,\overrightarrow {HC} } \right) + \left( {\overrightarrow {HC} ,\overrightarrow {HA} } \right) = \angle BHA + \angle BHC + \angle CHA = 2.\angle BHC = 2.\left( {{{180}^o} - {{100}^0}} \right) = {160^o}\)

Chọn  B.

Đáp án - Lời giải

Câu hỏi 25 :

Cho biết \(\cos \alpha  + \sin \alpha  = \frac{1}{3}.\) Giá trị của \(P = \sqrt {{{\tan }^2}\alpha  + {{\cot }^2}\alpha } \) bằng bao nhiêu?

  • A \(\frac{1}{4}\)              
  • B \(\frac{3}{4}\)              
  • C \(\frac{5}{4}\)              
  • D \(\frac{7}{4}\)

Đáp án: D

Phương pháp giải:

Áp dụng các công thức: \(\left\{ \begin{array}{l}1 + {\tan ^2}\alpha  = \frac{1}{{{\rm{co}}{{\rm{s}}^2}\alpha }};\,\,\,\,1 + {\cot ^2}\alpha  = \frac{1}{{{{\sin }^2}\alpha }}\\{\left( {\sin \alpha  + \cos \alpha } \right)^2} = 1 + 2\sin \alpha \cos \alpha \end{array} \right..\)

Lời giải chi tiết:

Ta có:  \(\cos \alpha  + \sin \alpha  = \frac{1}{3} \Rightarrow {\left( {\cos \alpha  + \sin \alpha } \right)^2} = \frac{1}{9} \Leftrightarrow 1 + 2\sin \alpha \cos \alpha  = \frac{1}{9} \Leftrightarrow \sin \alpha \cos \alpha  =  - \frac{4}{9}.\)

\(\begin{array}{l} \Rightarrow P = \sqrt {{{\tan }^2}\alpha  + {{\cot }^2}\alpha }  = \sqrt {\frac{1}{{{{\cos }^2}\alpha }} - 1 + \frac{1}{{{{\sin }^2}\alpha }} - 1}  = \sqrt {\frac{{{{\sin }^2}\alpha  + {{\cos }^2}\alpha }}{{{{\sin }^2}\alpha {{\cos }^2}\alpha }} - 2} \\\,\,\,\,\,\,\,\,\,\,\,\, = \sqrt {{{\left( {\frac{1}{{\sin \alpha \cos \alpha }}} \right)}^2} - 2}  = \sqrt {{{\left( { - \frac{9}{4}} \right)}^2} - 2}  = \frac{7}{4}.\end{array}\)

Chọn  D.

Đáp án - Lời giải

Câu hỏi 26 :

Cho biết \(\cos \alpha  =  - \frac{2}{3}.\) Giá trị của biểu thức \(P = \frac{{\cot \alpha  + 3\tan \alpha }}{{2\cot \alpha  + \tan \alpha }}\) bằng bao nhiêu?

  • A \(\frac{{ - 19}}{{13}}\)             
  • B \(\frac{{19}}{{13}}\)                
  • C \(\frac{{25}}{{13}}\)
  • D \(\frac{{ - 25}}{{13}}\)

Đáp án: B

Phương pháp giải:

Áp dụng các công thức: \(\tan \alpha  = \frac{{\sin \alpha }}{{\cos \alpha }};\,\,\,\cot \alpha  = \frac{{\cos \alpha }}{{\sin \alpha }};\,\,\,{\sin ^2}\alpha  + {\cos ^2}\alpha  = 1.\)

Lời giải chi tiết:

Ta có: \({\sin ^2}\alpha  + c{\rm{o}}{{\rm{s}}^2}\alpha  = 1 \Leftrightarrow {\sin ^2}\alpha  = 1 - c{\rm{o}}{{\rm{s}}^2}\alpha  = 1 - \frac{4}{9} = \frac{5}{9}\)

\( \Rightarrow P = \frac{{\frac{{{\rm{cos}}\alpha }}{{\sin \alpha }} + 3\frac{{\sin \alpha }}{{\cos \alpha }}}}{{2\frac{{\cos \alpha }}{{\sin \alpha }} + \frac{{\sin \alpha }}{{\cos \alpha }}}} = \frac{{{{\cos }^2}\alpha  + 3{{\sin }^2}\alpha }}{{2{{\cos }^2}\alpha  + {{\sin }^2}\alpha }} = \frac{{{{\left( { - \frac{2}{3}} \right)}^2} + 3.\frac{5}{9}}}{{2.{{\left( { - \frac{2}{3}} \right)}^2} + \frac{5}{9}}} = \frac{{19}}{{13}}.\)

Chọn  B

Đáp án - Lời giải

Câu hỏi 27 :

Cho tam giác \(ABC\). Tính giá trị của biểu thức:

\(S = \frac{{{{\sin }^3}\frac{{\angle B}}{2}}}{{{\rm{cos}}\left( {\frac{{\angle A + \angle C}}{2}} \right)}} + \frac{{{\rm{co}}{{\rm{s}}^3}\frac{{\angle B}}{2}}}{{\sin \left( {\frac{{\angle A + \angle C}}{2}} \right)}} - \frac{{\cos \left( {\angle A + \angle C} \right)}}{{\sin \angle B}}.\tan \angle B\)

  • A \(0\)      
  • B \(1\)
  • C \( - 1\)
  • D \(2\)

Đáp án: D

Phương pháp giải:

Áp dụng các công thức sau: \(\left\{ \begin{array}{l}{\sin ^2}\alpha  + c{\rm{o}}{{\rm{s}}^2}\alpha  = 1\\\tan \alpha .\cot \alpha  = 1 \Rightarrow \tan \alpha .\frac{{\cos \alpha }}{{\sin \alpha }} = 1\\\sin \left( {{{90}^0} - \alpha } \right) = c{\rm{os}}\alpha \\\cos \left( {{{180}^0} - \alpha } \right) =  - c{\rm{os}}\alpha \end{array} \right..\)

Lời giải chi tiết:

Ta có: \(\angle A + \angle B + \angle C = {180^o} \Rightarrow \angle A + \angle C = {180^o} - \angle B\)

Lại có: \(\sin \left( {\frac{{{{180}^o} - \angle B}}{2}} \right) = \cos \frac{{\angle B}}{2};\,\,\,\,\cos \left( {\frac{{{{180}^o} - \angle B}}{2}} \right) = \sin \frac{{\angle B}}{2}.\)

  \(\begin{array}{l} \Rightarrow S = \frac{{{{\sin }^3}\frac{{\angle B}}{2}}}{{\cos \left( {\frac{{\angle A + \angle C}}{2}} \right)}} + \frac{{{{\cos }^3}\frac{{\angle B}}{2}}}{{\sin \left( {\frac{{\angle A + \angle C}}{2}} \right)}} - \frac{{\cos \left( {\angle A + \angle C} \right)}}{{\sin \angle B}}.\tan \angle B\\\,\,\,\,\,\,\,\,\,\,\, = \frac{{{{\sin }^3}\frac{{\angle B}}{2}}}{{\cos \left( {\frac{{{{180}^o} - \angle B}}{2}} \right)}} + \frac{{{{\cos }^3}\frac{{\angle B}}{2}}}{{\sin \left( {\frac{{{{180}^o} - \angle B}}{2}} \right)}} - \frac{{\cos \left( {{{180}^o} - \angle B} \right)}}{{\sin \angle B}}.\tan \angle B\\\,\,\,\,\,\,\,\,\, = \frac{{{{\sin }^3}\frac{{\angle B}}{2}}}{{\sin \frac{{\angle B}}{2}}} + \frac{{{{\cos }^3}\frac{{\angle B}}{2}}}{{\cos \frac{{\angle B}}{2}}} - \frac{{ - \cos \angle B}}{{\sin \angle B}}.\tan \angle B\\\,\,\,\,\,\,\,\,\, = {\sin ^2}\frac{{\angle B}}{2} + {\cos ^2}\frac{{\angle B}}{2} + \cot \angle B.\tan \angle B = 1 + 1 = 2.\end{array}\)

Chọn  D

Đáp án - Lời giải

Câu hỏi 28 :

Cho tam giác \(ABC\) và ba số dương \(x,y,z\) thay đổi có tổng bình phương \({x^2} + {y^2} + {z^2} = {k^2},k \in \mathbb{R}.\) Giá trị lớn nhất của \(P = xy\cos \angle C + yz\cos \angle A + zx\cos \angle B\) ?

  • A \(\frac{{{k^2}}}{2}\)                
  • B \(\frac{{{k^2}}}{3}\)    
  • C \(\frac{k}{2}\)
  • D \(\frac{k}{3}\)

Đáp án: A

Phương pháp giải:

Với 3 vecto \(\overrightarrow a ,\,\,\overrightarrow b ,\,\,\overrightarrow c :\,\,{\left( {\overrightarrow a  + \overrightarrow b  + \overrightarrow c } \right)^2} \ge 0.\) 

\(\cos \left( {{{180}^0} - \alpha } \right) =  - \cos \alpha .\)

Lời giải chi tiết:

Đặt 3 vecto \(\overrightarrow {BX} ,\overrightarrow {CY} ,\overrightarrow {AZ} \) tương ứng là \(\overrightarrow x ,\,\,\overrightarrow y ,\,\,\overrightarrow z .\)

Ta có:

\(\begin{array}{l}{\left( {\overrightarrow x  + \overrightarrow y  + \overrightarrow z } \right)^2} \ge 0 \Leftrightarrow {x^2} + {y^2} + {z^2} + 2\overrightarrow x \overrightarrow y  + 2\overrightarrow y \overrightarrow z  + 2\overrightarrow z \overrightarrow x  \ge 0\\ \Leftrightarrow {k^2} + 2xy\cos \left( {{{180}^o} - \angle C} \right) + 2yz\cos \left( {{{180}^o} - \angle A} \right) + 2xz\cos \left( {{{180}^o} - \angle B} \right) \ge 0\\ \Leftrightarrow {k^2} - 2xy\cos \angle C - 2yz\cos \angle A - 2xz\cos \angle B \ge 0\\ \Leftrightarrow xy\cos \angle C + yz\cos \angle A + xz\cos \angle B \le \frac{{{k^2}}}{2}\end{array}\)

Vậy \({\rm{Max }}P{\rm{ = }}\frac{{{k^2}}}{2}\)

Chọn  A

Đáp án - Lời giải

Câu hỏi 29 :

Cho tam giác \(ABC.\) Tính \(P = \sin A.\cos \left( {B + C} \right) + \cos A.\sin \left( {B + C} \right)?\)

  • A \(P = 0\)
  • B \(P = 1\)
  • C \(P =  - 1\)        
  • D \(P =  2\)        

Đáp án: A

Phương pháp giải:

Nếu \(\alpha  + \beta  = {180^o}\) thì  \(\sin \alpha  = \sin \beta ;{\rm{ cos}}\alpha  =  - \cos \beta .\)

Lời giải chi tiết:

Giả sử  \(\angle A = \alpha ;\,\,\,\angle B + \angle C = \beta \)

\( \Rightarrow P = \sin A.\cos \left( {B + C} \right) + \cos A.\sin \left( {B + C} \right) = \sin \alpha \cos \beta  - \cos \alpha \sin \beta .\)

Trong tam giác \(ABC,\,\,\angle A + \angle B + \angle C = {180^o} \Rightarrow \alpha  + \beta  = {180^o}\)

\( \Rightarrow \sin \alpha  = \sin \beta ;{\rm{ cos}}\alpha  =  - \cos \beta .\)

Vậy \(P = \sin \alpha \cos \beta  + \cos \alpha \sin \beta  =  - \sin \alpha \cos \alpha  + \cos \alpha \sin \alpha  = 0.\) 

Chọn A.

Đáp án - Lời giải

Câu hỏi 30 :

Cho biết \(\cos \alpha  + \sin \alpha  = \frac{1}{3}.\) Giá trị của \(P = \sqrt {{{\tan }^2}\alpha  + {{\cot }^2}\alpha } \) bằng bao nhiêu?

  • A \(\frac{1}{4}\)              
  • B \(\frac{3}{4}\)              
  • C \(\frac{5}{4}\)              
  • D \(\frac{7}{4}\)

Đáp án: D

Phương pháp giải:

Áp dụng các công thức: \(\left\{ \begin{array}{l}1 + {\tan ^2}\alpha  = \frac{1}{{{\rm{co}}{{\rm{s}}^2}\alpha }};\,\,\,\,1 + {\cot ^2}\alpha  = \frac{1}{{{{\sin }^2}\alpha }}\\{\left( {\sin \alpha  + \cos \alpha } \right)^2} = 1 + 2\sin \alpha \cos \alpha \end{array} \right..\)

Lời giải chi tiết:

Ta có:  \(\cos \alpha  + \sin \alpha  = \frac{1}{3} \Rightarrow {\left( {\cos \alpha  + \sin \alpha } \right)^2} = \frac{1}{9} \Leftrightarrow 1 + 2\sin \alpha \cos \alpha  = \frac{1}{9} \Leftrightarrow \sin \alpha \cos \alpha  =  - \frac{4}{9}.\)

\(\begin{array}{l} \Rightarrow P = \sqrt {{{\tan }^2}\alpha  + {{\cot }^2}\alpha }  = \sqrt {\frac{1}{{{{\cos }^2}\alpha }} - 1 + \frac{1}{{{{\sin }^2}\alpha }} - 1}  = \sqrt {\frac{{{{\sin }^2}\alpha  + {{\cos }^2}\alpha }}{{{{\sin }^2}\alpha {{\cos }^2}\alpha }} - 2} \\\,\,\,\,\,\,\,\,\,\,\,\, = \sqrt {{{\left( {\frac{1}{{\sin \alpha \cos \alpha }}} \right)}^2} - 2}  = \sqrt {{{\left( { - \frac{9}{4}} \right)}^2} - 2}  = \frac{7}{4}.\end{array}\)

Chọn  D.

Đáp án - Lời giải

Xem thêm

>> Học trực tuyến Lớp 10 tại Tuyensinh247.com, Cam kết giúp học sinh học tốt, hoàn trả học phí nếu học không hiệu quả.